Keresés

Részletes keresés

Gergo73 Creative Commons License 2013.02.04 0 0 4671

Az Általad adott algoritmusok ugye nem ilyenek.

 

De ilyenek. Mindkét esetben van jó algoritmus, tehát készen vagyunk. Nekünk nem kell tudni, hogy melyik algoritmus a jó, az egyik mindenképpen jó. Az hogy a ZFC konzisztens vagy sem, az adva van "metamatikailag". Persze - ahogy elsőszülött mondja - a bizonyítás ugyanúgy elmondható ZFC-ben (annak ellenére, hogy Con(ZFC) független ZFC-től).

Előzmény: Nautilus_ (4658)
elsoszulott Creative Commons License 2013.02.04 0 0 4670

Meglep, hogy DancingerF "érti". A válasz ugyanis rossz.

 

 

Szerinted Gergő bizonyítása rossz a szokásos elsőrendű ZFC-ben, vagy formálisan jó, de valamilyen filozófiai okból rossznak tartod?

Előzmény: Nautilus_ (4658)
elsoszulott Creative Commons License 2013.02.04 0 0 4669

Pótlási axióma alatt a Replacement-et értem.

Boolos és Shoenfield foglalkoztak filozófiai szemszögből ezzel a dologgal. Shoenfield-nek van egy sztint-elmélete is erről.

Előzmény: Nautilus_ (4659)
elsoszulott Creative Commons License 2013.02.04 0 0 4668

A Grothendieck-halmazelmélet ereje ZFC+{létezik valódi osztálynyi erősen elérhetetlen}, bizonyára ezért tévesztetted össze.

 

 

"Amúgy ZFC+{létezik valódi osztálynyi erősen elérhetetlen számosság} már tudja modellezni MK-t." 

 

Ezt írtam és ez  igaz. Nincs összetévesztés.

Előzmény: Nautilus_ (4655)
Gergo73 Creative Commons License 2013.02.04 0 0 4667

hogy tudd, mielőtt tagadod

 

Köszönöm a tájékoztatást.

 

a Te világodban nagyon is attól igaz egy számelméleti állítás, hogy ZFC-tétel-e

 

Nem egészen. Ha a ZFC-ben eldönthető egy számelméleti állítás, akkor azt elfogadom igazságértéknek (igaz vagy hamis). Ez azért van, mert a ZFC-t nagyon megbízható eszköznek tartom. Ha a ZFC-ben nem dönthető el egy számelméleti állítás, akkor meg lehet próbálni egyéb érveket felhozni mellette vagy ellene. Pl. ha kiderülne az ikerprím-sejtésről vagy a Riemann-sejtésről, hogy az a ZFC-ben eldönthetetlen (feltével hogy ZFC konzisztens), akkor habozás nélkül elfogadnám őket igaznak (és szomorú lennék, hogy a ZFC ezt nem látja). Egyébként ha sok "fontos" állításról derülne ki, hogy ZFC-független, akkor a matematikusok előbb-utóbb lecserélnék valami másra, mert független állítások kusza halmazával nem nagyon lehet mit kezdeni.

Előzmény: Nautilus_ (4666)
Nautilus_ Creative Commons License 2013.02.04 0 0 4666

de az én világomban nem attól igaz vagy hamis egy számelméleti állítás (pl. hogy van-e végtelen sok ikerprím), hogy PA-ból levezethető-e.


Tudom. Én nem kívánlak meggyőzni, én tájékoztattalak az intuicionizmus egy filozófiai alapvetéséről, hogy tudd, mielőtt tagadod.

 

Mellesleg máskor szellemiségében mást mondasz: a Te világodban nagyon is attól igaz egy számelméleti állítás, hogy ZFC-tétel-e.

 

Előzmény: Gergo73 (4662)
Gergo73 Creative Commons License 2013.02.04 0 0 4665

Ha nem tesszük hozzá, akkor mindig minden ZFC-ben értendő. Az a hivatalos nyelv. Bocs.

Előzmény: Nautilus_ (4664)
Nautilus_ Creative Commons License 2013.02.04 0 0 4664

Nem tudsz köztes számosságot megadni ZFC-ben

 

Valóban nem, csakhogy Te nem ezt mondtad, hanem hogy úgy általában nem tudok megadni, ami nem igaz. 

 

Előzmény: Gergo73 (4661)
Nautilus_ Creative Commons License 2013.02.04 0 0 4663

(Gondold meg, hogy e tételek halmaza eldönthetetlen.

Ez azért lehet, mert a Gödel-számok halmaza felfut egy hipervéges halmazra, majd le a valódi végesekre.

 

E két mondatot felcseréltem véletlenül, és a zárójelet sem zártam le.:) 

 

Mindenesetre, arra kell gondolni, pontatlanul, de alapvetően helyesen fogalmazva, hogy a PA-tételeket generáló rekurzív függvény értékkészlete, azaz a Gödel-számok kiszámolása közben felfutunk hipervéges halmaz elemeire, majd vissza véges számokra, és így olyan Gödel-számot (is) kaphatunk, amely egy másik ZFC-modell PA-tételeket generáló rekurzív függvénye értékkészelétenek nem eleme.

 

Ez abból látható, hogy a PA-tételek halmazát a ZFC nem dönti el - sem bármilyen, rekurzív bővítése -, tehát lesz olyan adott modellbeli PA-tétel, amely PA-tétel-volta ZFC-független.

 

Előzmény: Nautilus_ (4660)
Gergo73 Creative Commons License 2013.02.04 0 0 4662

Ha az a kérdés, hogy egy phi Peano-tétel-e, eldönthetjük, ha tényleg az. Ekkor ugyanis levezethető a Peano-ból. De ha nem, akkor talán sosem tudhatjuk meg, hogy nem tétel, mert a Peano-tételek halmaza eldönthetetlen.

 

Rendben, de az én világomban nem attól igaz vagy hamis egy számelméleti állítás (pl. hogy van-e végtelen sok ikerprím), hogy PA-ból levezethető-e. A PA az egy lehetséges út a megismerésre, a ZFC egy másik és így tovább. Az eredeti kérdés (pl. hogy van-e végtelen sok ikerprím) számomra fontosabb és alapvetőbb, mint a PA vagy a ZFC, amit a vizsgálatára kifejlesztettünk vagy felhasználunk. Mondhatjuk úgy is, hogy én hiszek abban (avagy: abból indulok ki), hogy vagy van végtelen sok ikerprím, vagy nem, harmadik lehetőség nincs, és minden erőmmel (axiómák feltevésével, elméletek kifejlesztésével) azon vagyok, hogy eldöntsem, mi az igazság. Ebben a felfogásban nincs eldönthetetlen állítás: úgy alakítjuk a matematikát, hogy az eredetileg fontos kérdés eldönthető legyen végül. Persze itt fontos egymás meggyőzése, pl. ha közösen használni akarjuk a kiválasztási axiómát, akkor előtte közösen el kell fogadnunk azt "igaz"-nak.

Előzmény: Nautilus_ (4657)
Gergo73 Creative Commons License 2013.02.04 0 0 4661

Csak k "köztessége" ZFC-független.

 

Jó, ez világos, de ezzel nem mondtunk sokat. A kontinuumhipotézis már a megfogalmazásában is a rákövetkező számosságról szól, nevezetesen hogy az kisebb-e a hatványszámossággal vagy sem.

 

Kifejtenéd ezt?

 

Igazából nincs mit nagyon kifejtenem. A ZF és a ZFC a jelenleg általánosan elfogadott fogalomrendszert és intuíciót tükrözi, és ebben a halmazelméletben nem lehet megragadni (avagy nyakon csípni) köztes számosságot. Felteheted a létezését, de ez csak olyan, mint a levegőbe lövöldözés. Ezzel persze nem azt mondom, hogy más halmazelméletek nem érdekesek, csak én az általánosan elfogadott halmazelméletről beszéltem végig. Amúgy a kérdés kapcsán olvastam a Wikipédiában: "Gödel believed that CH is false and that his proof that CH is consistent only shows that the Zermelo-Fraenkel axioms do not adequately describe the universe of sets."

 

Számos példát írhatnék még, amikor köztes számosságot adtak meg, definiáltak, és ez fontos volt.

 

Nem tudsz köztes számosságot megadni ZFC-ben, hiszen konzisztens ZFC-vel, hogy ilyen nincs. Persze felteheted, hogy van köztes számosság (pl. feltehetsz adalék axiómákat, amiből ilyen számosság létezése következik), de ez nem az, amiről beszéltem.

Előzmény: Nautilus_ (4656)
Nautilus_ Creative Commons License 2013.02.04 0 0 4660

Még egy dolog... Mivel a Turing gépek halmazok, vannak a kiszámíthatóságnak nemsztenderd modelljei? Ha vannak, van ezeknek valami következménye, foglalkozik ilyesmivel valaki?

 

Igen, vannak.

Vannak ZFC-modellek, amelyek nem omega-modellek, azaz nem a sztenderd modellt látják sztenderdnek. Ez úgy lehet, hogy pl. ~Con(ZFC) igaz a ZFC-modellben, miközben a ZFC konzisztens. Hogyan lehet ez? Úgy, hogy az ellentmondás levezetése nemsztenderd, azaz hipervéges.

 

Ha ilyen, N* "pszeudo-sztenderd" modell van egy ZFC-modellben, akkor ha kiszámolsz valamit a modell elméletével, átformalizálva, olyan Turing-gépet használsz, amely hipersok lépést is tehet. Hiszen a Turing-gép csak egy függvény a természetes számokon - amelyek, itt, végtelenül nagyok is lehetnek.

 

Igen, foglalkoznak ilyennel. Ez ugyanis egy hipervéges logikához vezet, amely nem effektív ugyan többnyire, de azért mindenféle tulajdonságai vannak.

 

Néha viszont effektív: kedvenc példám, hogy egy ilyen modell elméletében olyan PA-tétel is lehet (tehát PA|-phi formula Gödel-száma), amely más modellben PA-független. (Gondold meg, hogy e tételek halmaza eldönthetetlen.

Ez azért lehet, mert a Gödel-számok halmaza felfut egy hipervéges halmazra, majd le a valódi végesekre.

 

Előzmény: DancingerF (4624)
Nautilus_ Creative Commons License 2013.02.04 0 0 4659

Ezt sosem értettem, hogy a Ptótlási axiómát hogyan motiválja a cummulativ hierarchia

 

Úgy, hogy a Hatványhalmaz-operáció intuitíve "minden lehetséges" halmazt elkészít a hierarchiában. Ezért - intuitíve ismét - a Hierarchiában a Pótlási is igaz lesz.

 

Előzmény: elsoszulott (4632)
Nautilus_ Creative Commons License 2013.02.04 0 0 4658

Szerintem félreérted az algoritmus fogalmát, vagy esetleg rosszul teszed fel a kérdést. Hadd teszteljelek: van-e olyan algoritmus, ami azt írja ki, hogy "ZFC konzisztens" vagy "ZFC inkonzisztens" aszerint, hogy ZFC konzisztens vagy ZFC inkonzisztens?

 

Pedig a válasz egyszerű. Triviálisan van olyan algoritmus, amit azt írja ki, hogy "ZFC konzisztens", és triviálisan van olyan is, ami azt írja ki, hogy "ZFC inkonzisztens". Tehát bármi legyen is az igazság (a két lehetőség közül), van olyan algoritmus, ami azt írja ki.

 

 

Meglep, hogy DancingerF "érti". A válasz ugyanis rossz.

Triviálisan van algoritmus, amely minden véges sztringet kiír, csakhogy a Te kérdésed az volt, hogy "aszerint, hogy ZFC konzisztens, vagy...". Az Általad adott algoritmusok ugye nem ilyenek.

 

Ahhoz, hogy ilyen legyen, kellene egy - alapvetően metafizikai-ismeretelméleti okból elfogadott program az univerzális gépre, amely plauzibilisen következtet ZFC konzisztenciájára, vagy az ellenkezőjére. Mondjuk, ha az erősen elérhetetlen számosságot elfogadnánk, mély megfontolásból, az elegendő volna (mert következik belőle ZFC-ben Con(ZFC).

De vannak más, matematikán kívüli érvelések is.

 

Minden, amit a matematikában mondunk, hit kérdése. Még a formulaindukció, ez a rendkívül evidens eljárás is a végtelenre tett kikötés, amelyet nyilvánvalóként elfogadunk.

 

Előzmény: Gergo73 (4636)
Nautilus_ Creative Commons License 2013.02.04 0 0 4657

Illetve számomra tényleg nem kérdéses, hogy ZFC vagy konzisztens vagy nem, az én világomba (fogalomrendszerembe) nem fér bele egy harmadik lehetőség.

 

 

Először is, szögezzük le, hogy a konstruktivizmus egy gyűjtőcsoport, azaz sokféle létezik.

Az intuicionizmust szokás konstruktivizmusnak tekinteni, én támogatom ezt az elképzelést.

 

------

 

Hadd magyarázzam meg, miért tagadja Brouwer (Weyl, Aczel, Dummett...) a Kizárt Harmadik elvét.

Filozófiai okokból. Ők úgy gondolták, hogy a matematikai igazságnak a szubjektum (intuíció) számára megismerhetőnek kell lennie. Vegyük most Gödel tételét (bár ez Brouwer számára ismeretlen volt 1931-ig; inkább pl. Cantor tételével, a hatványhalmaz-axiómával érveltek, vagy az analízisből merítettek).

 

Ha az a kérdés, hogy egy phi Peano-tétel-e, eldönthetjük, ha tényleg az. Ekkor ugyanis levezethető a Peano-ból. De ha nem, akkor talán sosem tudhatjuk meg, hogy nem tétel, mert a Peano-tételek halmaza eldönthetetlen. És ha az, akkor a szubjektum - és az intuíció - számára igazsága hozzáférhetetlen.

Ezért mondjuk azt, hogy (phi v ~phi) nem igaz, mert úgy gondolták, hogy ha phi független, akkor sem nem igaz, sem nem hamis - nincs eljárásunk phi igazságértékének megállapítására. Tehát a matematikában e tautológia nem érvényes.

 

Ebből még az is következik, hogy az N sztenderd modellen sem lehet minden formula igaz, vagy hamis - más modellelmélet kell. Először ez volt az Heyting-algebra, majd a Kripke-(Beth-Joyal)-szemantika (1966.), és van topologikus szemantika is (Tarski-McKinsey, azt hiszem). 

 

Előzmény: Gergo73 (4643)
Nautilus_ Creative Commons License 2013.02.04 0 0 4656

Gödel munkája alapján a jelenlegi fogalmaink szerint nem lehet köztes számosságot konstruálni vagy felmutatni, tehát praktikusan ilyen köztes számosság nem létezik. Szóval az eredeti kontinuumhipotézisre én azt mondanám, hogy az "igaz a valóságban", vagy a "gyakorlatban".

 

Kifejtenéd ezt?

Gödel igazolta, hogy a ZFC-vel konzisztens, hogy igaz a CH. Tehát azt igazolta, hogy vagy ZFC-független, vagy ZFC-tétel. De ez nem jelenti, hogy "bizonyos esetekben" nem lehet konstruálni köztes k számosságot, még a definíciója is ZFC-ben lehet (azaz, k egyértelműen létezhet ZFC-ben).

 

Csak k "köztessége" ZFC-független.

 

A Martin-axióma igazsága (egy Aleph_n-re) esetén például számos köztes számosság lehet, és ez az axióma számos "praktikus" feladatot old meg az általános topológiában, végtelen kombinatorikában. A Solovay-modellben minden valóshalmaz Lebesgue-mérhető, és ~CH.

Számos példát írhatnék még, amikor köztes számosságot adtak meg, definiáltak, és ez fontos volt.

 

Előzmény: Gergo73 (4646)
Nautilus_ Creative Commons License 2013.02.04 0 0 4655

Amúgy ZFC+{létezik valódi osztálynyi erősen elérhetetlen számosság} már tudja modellezni MK-t.

 

ZFC+{létezik egyetlen erősen elérhetetlen} elegendő, de nem szükséges: P(V_{erősen elérhetetlen})|=MK.

Nem ismert, tudtommal, olyan pontos elsőrendű feltétel, amely ZFC mellett éppen ekvikonzisztens a Morse-Kelley-vel. Van megszámlálható MK-modell, de ez nem jelenti, hogy abban - vagy bármelyikben - van ZFC+{erősen elérhetetlen}-modell.

 

A Grothendieck-halmazelmélet ereje ZFC+{létezik valódi osztálynyi erősen elérhetetlen}, bizonyára ezért tévesztetted össze.

 

Előzmény: elsoszulott (4639)
Nautilus_ Creative Commons License 2013.02.04 0 0 4654

Hát hogyan kell/lehet formalizálni egy olyan eljárást, amelyikben lehet, hogy nagyon nagyon sokszor (mondjuk kontinuum sokszor) kell összeadni.

 

 

Van ilyen eljárás, pontosabban olyan, ahol az intuíció végtelen, sőt, kontinuum-sok összeadást éreztet velünk. Először is, ilyen az integrál fogalma. Itt a (Lebesgue esetben) már eleve megszámlálható sorösszegek limesze (kontinuum számosságú?) "összeadás".

 

Azután, ha veszel egy topologikus vektorteret, annak egy bázisát, az lehet olyan, hogy egy testből (ami felett a vektortér van) vett "skalárokkal" szorozva elemeit pl. kontinuum (vagy akármennyi számosságú) sok "összeadást" (szumma jellel, mint konvencióval) írunk fel ahhoz, hogy minden vektort előállítsunk. Hilbert-tér is lehet ilyen (kontinuum dimenziós).

 

Az algebrában mindig csak véges sok változós művelet van. Ezt azonban sokszor általánosíthatod végtelenre, egyszerűen, vagy bonyolultabban posztulálva vagy definiálva a végtelen változós művelet eredményét. (például lezárási operátorral, vagy úgy, hogy igazolod, hogy egy definíción belül maradsz az általánosítással is)

Ez értelmesség, azaz konzisztencia kérdése.

Olyan esetekben, amikor végtelen változós művelet van, lényegében mindig topológiát is definiálunk. Ez azért van, mert a művelet kezdőszeletei "konvergálnak" a végtelen művelettel előállított elemhez, és a konvergencia értelmezése algebrai struktúrán: topológia.

 

 

Előzmény: DancingerF (4649)
elsoszulott Creative Commons License 2013.02.03 0 0 4653

Valós mérhető számosságoknál például kellhet kontinuum vagy mégtöbb nemnegatív valóst összeadni, ez akkor értelmes ha csak megszámlálható sok nem 0 van közte, ilyenkor meg mivel tetszőleges sorrendben összeadhatod a szokásos módon.

Előzmény: DancingerF (4649)
Gergo73 Creative Commons License 2013.02.03 0 0 4652

Nem értem, mit kell néznem pár sorral lejjebb. Ird le pontosan, hogy miféle eljárásról beszélsz, és miért kellene formalizálni és hol hallottál róla.

Előzmény: DancingerF (4651)
DancingerF Creative Commons License 2013.02.02 0 0 4651

Lásd pár sorral lentebb; mert érdekes; Nem értek veled egyet (persze ez nem sokat nyom a latba egy matematikus véleménye ellen)

Előzmény: Gergo73 (4650)
Gergo73 Creative Commons License 2013.02.02 0 0 4650

Nem ismerek ilyen eljárást, és nem értem miért kellene formalizálni. Én matematikus vagyok, a matematikában pedig eleve minden rendesen formalizálva van.

Előzmény: DancingerF (4649)
DancingerF Creative Commons License 2013.02.02 0 0 4649

Hát hogyan kell/lehet formalizálni egy olyan eljárást, amelyikben lehet, hogy nagyon nagyon sokszor (mondjuk kontinuum sokszor) kell összeadni.

Előzmény: Gergo73 (4648)
Gergo73 Creative Commons License 2013.02.02 0 0 4648

Mire gondolsz? Nem értem a kérdést.

Előzmény: DancingerF (4647)
DancingerF Creative Commons License 2013.02.02 0 0 4647

És akkor hogyan kell értelmezni egy ilyen végtelen eljárást?

Gergo73 Creative Commons License 2013.02.01 0 0 4646

Vártam ezt a kérdést. Ha muszáj válaszolnom, akkor különválasztanám az eredeti kontinuumhipotézist és az általánosított kontinuumhipotézist. Az eredeti kontinuumhipotézis két elég konkrét számosságról szól, a természetes számokról és a hatványhalmazáról. Gödel munkája alapján a jelenlegi fogalmaink szerint nem lehet köztes számosságot konstruálni vagy felmutatni, tehát praktikusan ilyen köztes számosság nem létezik. Szóval az eredeti kontinuumhipotézisre én azt mondanám, hogy az "igaz a valóságban", vagy a "gyakorlatban". Persze ez nem zárja ki azt, hogy a fogalmainkat, az intuíciónkat úgy módosítsuk a jövőben, hogy legyen köztes számosság. Cohen munkáját egy pozitív lépésnek tekinthetjük ebbe az irányba: ő kontruált olyan ZFC-modellt, amiben a köztes számosság létezik. Az általánosított kontinuumhipotézis kicsit más tészta, mert az eleve olyan számosságokról is szól, amik nem definiálhatók pl. ZFC-ben, tehát nem lehet rájuk olyan konkrétan gondolni, mint a természetes számokra vagy a hatványhalmazára.

Előzmény: elsoszulott (4645)
elsoszulott Creative Commons License 2013.02.01 0 0 4645

A kontinuumhipotézisről is így vélekedsz?

Előzmény: Gergo73 (4643)
Gergo73 Creative Commons License 2013.01.31 0 0 4644

A nemkonstruktív bizonyításra itt egy jobb példa (Iwaniec után szabadon):

 

Tétel. Van olyan n természetes szám, hogy ha n-ig igaz a Goldbach-sejtés, akkor igaz a Goldbach-sejtés.

 

Bizonyítás. Ha igaz a Goldbach-sejtés, akkor minden n jó. Ha hamis a Goldbach-sejtés, akkor a legkisebb ellenpélda választható n-nek. QED

 

Megjegyzem, hogy ez egy teljesen korrekt bizonyítás (az elsőrendű logikában, tehát ahogy a matematikus értik a szót). Ez még akkor is így van, ha a Goldbach-sejtés független a ZFC-től (amit amúgy nem hiszek).

Előzmény: GosuMicroFTW (4640)
Gergo73 Creative Commons License 2013.01.31 0 0 4643

Amúgy szerintem Gergő elégedett azzal, hogy a bizonyításai, a szokásos elsőrendű ZFC-ben bizonyítások legyenek és másféle logikai rendszerek nem igazán érdeklik.

 

Valóban így van. Illetve számomra tényleg nem kérdéses, hogy ZFC vagy konzisztens vagy nem, az én világomba (fogalomrendszerembe) nem fér bele egy harmadik lehetőség. Hasonlóan nem kérdéses számomra, hogy az ikerprím-sejtés vagy igaz vagy nem.

Előzmény: elsoszulott (4642)
elsoszulott Creative Commons License 2013.01.31 0 0 4642

Szerintem Gergő lentebbi érvelését egy intuicionista nem fogadná el.

 

Gergő feltételezte, hogy a "ZFC konzisztens " és "ZFC nem konzisztens" állítások közül valamelyik igaz. Mint ahogy én gyakran feltételezem, hogy egy gráfban vagy van kör vagy nincs. Ezek az intuicionista logikában nem megengedett feltevések.

 

Amúgy szerintem Gergő elégedett azzal, hogy a bizonyításai, a szokásos elsőrendű ZFC-ben bizonyítások legyenek és másféle logikai rendszerek nem igazán érdeklik.

Előzmény: GosuMicroFTW (4640)

Ha kedveled azért, ha nem azért nyomj egy lájkot a Fórumért!